Difference between revisions of "2025 AIME II Problems/Problem 5"
m |
(→Solution) |
||
(5 intermediate revisions by 3 users not shown) | |||
Line 1: | Line 1: | ||
== Problem == | == Problem == | ||
− | Suppose <math>\triangle ABC</math> has angles <math>\angle BAC = 84^\circ, \angle ABC=60^\circ,</math> and <math>\angle ACB = 36^\circ.</math> Let <math>D, E,</math> and <math>F</math> be the midpoints of sides <math>\overline{BC}, \overline{AC},</math> and <math>\overline{AB},</math> respectively. The circumcircle of <math>\triangle DEF</math> intersects <math>\overline{BD}, \overline{AE},</math> and <math>\overline{AF}</math> at points <math>G, H,</math> and <math>J,</math> respectively. The points <math>G, D, E, H, J,</math> and <math>F</math> divide the circumcircle of <math>\triangle DEF</math> into six minor arcs, as shown. Find <math>\widehat{DE}+2\cdot \widehat{HJ} + 3\cdot \widehat{FG},</math> where the arcs are measured in degrees. | + | Suppose <math>\triangle ABC</math> has angles <math>\angle BAC = 84^\circ, \angle ABC=60^\circ,</math> and <math>\angle ACB = 36^\circ.</math> Let <math>D, E,</math> and <math>F</math> be the midpoints of sides <math>\overline{BC}, \overline{AC},</math> and <math>\overline{AB},</math> respectively. The circumcircle of <math>\triangle DEF</math> intersects <math>\overline{BD}, \overline{AE},</math> and <math>\overline{AF}</math> at points <math>G, H,</math> and <math>J,</math> respectively. The points <math>G, D, E, H, J,</math> and <math>F</math> divide the circumcircle of <math>\triangle DEF</math> into six minor arcs, as shown. Find <math>\widehat{DE}+2\cdot \widehat{HJ} + 3\cdot\widehat{FG},</math> where the arcs are measured in degrees. |
<asy> | <asy> | ||
import olympiad; | import olympiad; | ||
Line 23: | Line 23: | ||
== Solution == | == Solution == | ||
− | Notice that due to midpoints, <math>\triangle DEF\ | + | Notice that due to midpoints, <math>\triangle DEF\sim\triangle FBD\sim\triangle AFE\sim\triangle EDC\sim\triangle ABC</math>. As a result, the angles and arcs are readily available. Due to inscribed angles, |
<cmath>\widehat{DE}=2\angle DFE=2\angle ACB=2\cdot36=72^\circ</cmath> | <cmath>\widehat{DE}=2\angle DFE=2\angle ACB=2\cdot36=72^\circ</cmath> | ||
Similarly, | Similarly, | ||
Line 30: | Line 30: | ||
In order to calculate <math>\widehat{HJ}</math>, we use the fact that <math>\angle BAC=\frac{1}{2}(\widehat{FDE}-\widehat{HJ})</math>. We know that <math>\angle BAC=84^\circ</math>, and | In order to calculate <math>\widehat{HJ}</math>, we use the fact that <math>\angle BAC=\frac{1}{2}(\widehat{FDE}-\widehat{HJ})</math>. We know that <math>\angle BAC=84^\circ</math>, and | ||
<cmath>\widehat{FDE}=360-\widehat{FE}=360-2\angle FDE=360-2\angle CAB=360-2\cdot84=192^\circ</cmath> | <cmath>\widehat{FDE}=360-\widehat{FE}=360-2\angle FDE=360-2\angle CAB=360-2\cdot84=192^\circ</cmath> | ||
+ | |||
Substituting, | Substituting, | ||
− | |||
− | |||
− | |||
− | Thus, <math>\widehat{DE}+2\cdot\widehat{HJ}+3\cdot\widehat{FG}=72+48+216=\boxed{336}^\circ</math>. ~eevee9406 | + | \begin{align*} |
+ | 84 &= \frac{1}{2}(192-\widehat{HJ}) \\ | ||
+ | 168 &= 192-\widehat{HJ} \\ | ||
+ | \widehat{HJ} &= 24^\circ | ||
+ | \end{align*} | ||
+ | |||
+ | Thus, <math>\widehat{DE}+2\cdot\widehat{HJ}+3\cdot\widehat{FG}=72+48+216=\boxed{336}^\circ</math>. | ||
+ | |||
+ | ~ [https://artofproblemsolving.com/wiki/index.php/User:Eevee9406 eevee9406] | ||
+ | |||
+ | ~ Edited by [https://artofproblemsolving.com/wiki/index.php/User:Aoum aoum] | ||
+ | |||
+ | Alternatively, | ||
+ | |||
+ | \begin{align*} \widehat{HJ} &= \widehat{FH} + \widehat{JE} - \widehat{FE} \\ &= 2\angle FEH + 2\angle JFE - 2\angle FDE \\ &= 2 \cdot 36^\circ + 2 \cdot 60^\circ - 2 \cdot 84^\circ \\ &= 24^\circ. \end{align*} | ||
+ | |||
+ | ~ [https://artofproblemsolving.com/community/user/1096232 Pengu14] | ||
==See also== | ==See also== |
Latest revision as of 23:17, 17 February 2025
Problem
Suppose has angles
and
Let
and
be the midpoints of sides
and
respectively. The circumcircle of
intersects
and
at points
and
respectively. The points
and
divide the circumcircle of
into six minor arcs, as shown. Find
where the arcs are measured in degrees.
Solution
Notice that due to midpoints, . As a result, the angles and arcs are readily available. Due to inscribed angles,
Similarly,
In order to calculate , we use the fact that
. We know that
, and
Substituting,
\begin{align*} 84 &= \frac{1}{2}(192-\widehat{HJ}) \\ 168 &= 192-\widehat{HJ} \\ \widehat{HJ} &= 24^\circ \end{align*}
Thus, .
~ Edited by aoum
Alternatively,
\begin{align*} \widehat{HJ} &= \widehat{FH} + \widehat{JE} - \widehat{FE} \\ &= 2\angle FEH + 2\angle JFE - 2\angle FDE \\ &= 2 \cdot 36^\circ + 2 \cdot 60^\circ - 2 \cdot 84^\circ \\ &= 24^\circ. \end{align*}
~ Pengu14
See also
2025 AIME II (Problems • Answer Key • Resources) | ||
Preceded by Problem 4 |
Followed by Problem 6 | |
1 • 2 • 3 • 4 • 5 • 6 • 7 • 8 • 9 • 10 • 11 • 12 • 13 • 14 • 15 | ||
All AIME Problems and Solutions |
The problems on this page are copyrighted by the Mathematical Association of America's American Mathematics Competitions.